Problem 3! IMO 2015

Let \(ABC\) be an acute triangle with \(AB > AC\). Let \(\Gamma\) be its cirumcircle, HH its orthocenter, and FF the foot of the altitude from AA. Let MM be the midpoint of BCBC. Let QQ be the point on Γ\Gamma such that HQA=90\angle HQA = 90^{\circ} and let KK be the point on Γ\Gamma such that HKQ=90\angle HKQ = 90^{\circ}. Assume that the points AA, BB, CC, KK and QQ are all different and lie on Γ\Gamma in this order.

Prove that the circumcircles of triangles KQHKQH and FKMFKM are tangent to each other.

This is part of the set IMO 2015
#Geometry #Circumcenter #Orthocenter #TangentOfCircles #Circumcircles

Note by Sualeh Asif
5 years, 11 months ago

No vote yet
1 vote

  Easy Math Editor

This discussion board is a place to discuss our Daily Challenges and the math and science related to those challenges. Explanations are more than just a solution — they should explain the steps and thinking strategies that you used to obtain the solution. Comments should further the discussion of math and science.

When posting on Brilliant:

  • Use the emojis to react to an explanation, whether you're congratulating a job well done , or just really confused .
  • Ask specific questions about the challenge or the steps in somebody's explanation. Well-posed questions can add a lot to the discussion, but posting "I don't understand!" doesn't help anyone.
  • Try to contribute something new to the discussion, whether it is an extension, generalization or other idea related to the challenge.
  • Stay on topic — we're all here to learn more about math and science, not to hear about your favorite get-rich-quick scheme or current world events.

MarkdownAppears as
*italics* or _italics_ italics
**bold** or __bold__ bold

- bulleted
- list

  • bulleted
  • list

1. numbered
2. list

  1. numbered
  2. list
Note: you must add a full line of space before and after lists for them to show up correctly
paragraph 1

paragraph 2

paragraph 1

paragraph 2

[example link](https://brilliant.org)example link
> This is a quote
This is a quote
    # I indented these lines
    # 4 spaces, and now they show
    # up as a code block.

    print "hello world"
# I indented these lines
# 4 spaces, and now they show
# up as a code block.

print "hello world"
MathAppears as
Remember to wrap math in \( ... \) or \[ ... \] to ensure proper formatting.
2 \times 3 2×3 2 \times 3
2^{34} 234 2^{34}
a_{i-1} ai1 a_{i-1}
\frac{2}{3} 23 \frac{2}{3}
\sqrt{2} 2 \sqrt{2}
\sum_{i=1}^3 i=13 \sum_{i=1}^3
\sin \theta sinθ \sin \theta
\boxed{123} 123 \boxed{123}

Comments

I can imagine that inversion kills this problem, here's my solution without using that, I don't expect many to understand this at this point as I wrote it in a hurry.

Let II denote the midpoint of HQHQ. It suffices to show that the circumcenter of KFMKFM lies on IKIK, which equates to proving IKM=90KFC=KFH\angle IKM=\angle 90-\angle KFC=\angle KFH.

AH,QH,KHAH,QH,KH are extended to meet (O)(O) again at E,Z,XE,Z,X respectively. It is well known and obvious that OQX,AZ,MQZO\in QX, AZ, M\in QZ. Let FF' denote the midpoint of XHXH, AF(O)=YAF'\cap (O)=Y.

Since (XZMF)(HQ(XZ\parallel MF')\perp (HQ, therefore XZMFAQXZ\parallel MF'\parallel AQ. By the converse of Pascal's theorem, we can deduce that AFKM=YAF'\cap KM=Y. Note that AHXFKHEF\triangle AHXF'\sim \triangle KHEF, thus FKE=XAF=XKY\angle FKE=\angle XAF'=\angle XKY. Additionally, IKH=OKA=AZK=AEK\angle IKH=\angle OKA=\angle AZK=\angle AEK by spiral similarity about KK between (I),(O)(I),(O). Hence HFK=AEK+FKE=IKH+XKY=IKM\angle HFK=\angle AEK+\angle FKE=\angle IKH+\angle XKY=\angle IKM and we are done.

Xuming Liang - 5 years, 11 months ago

I don't know if that works or not, but try to use Nine-point circle to prove it. And I think that the point KK is probably Feuerbach point of some triangle.

Samuraiwarm Tsunayoshi - 5 years, 11 months ago
×

Problem Loading...

Note Loading...

Set Loading...